2. The direct distance between city A and city B is 200 miles. The direct distance
between city B and city C is 300 miles. Which could be the direct distance between city
C and city A?
1) 50 miles
2) 350 miles
3) 550 miles
4) 650 miles

2. The Direct Distance Between City A And City B Is 200 Miles. The Direct Distancebetween City B And

Answers

Answer 1

Answer:

2

Step-by-step explanation:


Related Questions

Expand and simplify (3x+4)(2x+3)

Answers

Answer:

72 you just need to keep multiplying

Step-by-step explanation:

3x4. 2x3

12 x 6

72

The value of log₃⁵ x log₂₅⁹ is...

Answers

Answer:

i dont know this answer but the answer will be 2log3 ok cheak it clearly wheather it is right or wrong

Which statement accurately describes the difference between short-term and long-term capital gains in terms of taxes? A. Long-term capital gains are from investments that have been held for more than one year and are taxed at a lower rate than short-term capital gains. B. Long-term capital gains are from investments that have been held for at least six months and are taxed at a lower rate than short-term capital gains. C. Long-term capital gains are from investments that have been held for more than one year and are taxed at a higher rate than short-term capital gains. D. Long-term capital gains are from investments that have been held for at least six months and are taxed at a higher rate than short-term capital gains.

Answers

Answer: A. Long-term capital gains are from investments that have been held for more than one year and are taxed at a lower rate than short-term capital gains.

Step-by-step explanation:

Long-term capital gains are indeed from investments that have been held for more than a year as opposed to short term gains from investments of less than a year. This means that long term gains can come from investments such as stocks and bonds.

Long-term capital gains are taxed at a lower rate than short term gains with most long term gains being taxed at 15% or lower. This is in contrast to short-term gains that are taxed at the same rate as ordinary income which means it could go up to as high as 37%.

Answer: A

Step-by-step explanation:

A scale drawing of Elizabeth's rectangular room is 4 inches by 7 inches. The scale is 1 inch = 3 feet. What is the area, in square feet, of Elizabeth's room?

Answers

Answer:

84 feet

Step-by-step explanation:

4 inches× 7 inches=28inches

28 inches×3feet=84feet

1. Maggie currently has a balance of $-15 in her bank account, she deposits $11
into her account. How much money does Maggie have in her account? *

Answers

-15+11=-4 so Maggie has -4 in her account

Answer: Maggie now has $-4 in her bank account.


Explanation: The word “deposits” usually means “add”, so all you need to do is:

-15+11
=-4

Therefore, Maggie now has a balance of $-4 in her bank account.


Hope this helps シ

Q5. A surprise Mathematics test was conducted in a class of 12 students. The
results are shown below.
45
22 12
9 6 63
63 78
12
69 81 3 19
4 points
Calculate the mean score
O 34.92
O 457
O 20.5
O 22​

Answers

Answer:

Mean value of score = 40.167 (Approx.)

Step-by-step explanation:

Given:

Number of students in class = 12

Numbers;

45 , 22, 12,  96, 63,  63, 78 , 12 , 69, 81, 3, 19

Find:

Mean value of score

Computation:

Mean = Sum of all observation / Number of students

Mean value of score = [45 + 22 + 12 +  96 + 63 +  63 + 78  + 12 + 69 + 81 + 3 + 19] / 12

Mean value of score = [482] / 12

Mean value of score = 40.167 (Approx.)

What is the inverse of the function f(x) = 169x2 when x > 0?

Answers

Answer:

y = (√x)/13 for x > 0

Step-by-step explanation:

Actually, this is f(x) = 169x^2; the " ^ " indicates exponentiation.  The graph consists of the right-hand half of the parabola 169x^2.  Before we attempt to find the inverse function, we determine whether or not this function passes the vertical line test.  If x is restricted to (0, infinity), a horizontal line intersects this graph only once.  Therefore, this function has an inverse.

1.  Replace 'f(x)' with 'y:'  y = 169x^2

2.  Interchange x and y:  x = 169y^2

3.  Solve for y:  y^2 = x/169, or y = (√x)/13 for x > 0

1. Explain how to find the area of a complex figure. Draw an example to help illustrate
your understanding. Please help!

Answers

Answer:

Refer to the attached file

Hope it helps..

A computer store bought a program at a cost of ​$10 and sold it at a selling price of ​$13. Find the percent markup.

Answers

Answer: 30 %

Step-by-step explanation:
Cost = $10
Selling price = $13
Markup = selling price - cost price
= $13 - $10
= $3
percent markup = markup/cost
= $3/$10
= 0.3 = 30%
Percentage markup is 30

If we have a ratio of 5:1 boys to girls in the class and there are only 5 girls in the class, How many boys are there in the class?

Answers

Answer:

25

Step-by-step explanation:

5x5=25

Hope this helps! :)

Answer:

25 boys

Explanation:

Because if there are 5 girls in the class then that means that you have to multiply the ratio of 5:1 by 5, which equals 25:5, meaning there’s 25 boys in the class.

Evaluate 5(x-1)-2when x=3

Answers

Answer:

3

Step-by-step explanation:

5(x-1)-2

5(2-1)-2

5×1-2

5-2

3

Hope this helps you :)

please mark brainliest

5(x-1)-2
5(3-1)-2
5(2)-2
10-2
8

6.) Find the square root(s)

Answers

Answer:

the square root is 0.08/8

-2/5 or -0.4
Is the square root

There is a special offer at the book shop.
Ryan chooses 4 books that cost
£4.00
£3.80
£3.00
and £1.20
Special
offer
4 for the
price of 3.
Buy any 4 books and get
the cheapest one completely free!
What fraction of the normal total price does he save?
Choose the correct calculation.
£4.00 + £3.80 + £3.00 + £1.20
£1.20
£1.20
£4.00 + £3.80 + £3.00 + £1.20
£1.20
£4.00 + £3.80 + £3.00
x (£4.00 + £3.80 + £3.00 + £1.20)

Answers

price of 3Step-by-step explanation:

Find the area of the figure below

Answers

Answer:

521

Step-by-step explanation:

Can someone help and explain how to do these problems please

Answers

Step-by-step explanation:

1st one right angle

Right angle is of 90°

and 70° is given so subtract them

90-70=30 so y is 30°

2nd one

35° given and

[tex] \frac{35}{7} = 5[/tex]

so 7x5=35

t=5

What is the relationship between the values tand s plotted on the number line
below?
S
A. t= s
B. t>s
O c. tcs

Answers

Answer:

The answer should be t > s. (B) is the answer.

16p⁷q⁸÷2p⁴q⁵
whats the answer?

Answers

Answer:

[tex]8 {p}^{3} {q}^{3} [/tex]

Step-by-step explanation:

Hope it is helpful....

what is 0.06 divided by 8 working out

Answers

It’s an answer for your question

Answer:

0.0075

Step-by-step explanation:

 0. 0 0 7 5

_______________________

8  ÷ 0. 0 6 0 0

− 0        

_______________________

 0 0      

−   0      

_______________________

   0 6    

 −   0    

_______________________

    6 0  

   − 5 6  

_______________________

      4 0

     − 4 0

_______________________

         0

If my answer is incorrect, pls correct me!

If you like my answer and explanation, mark me as brainliest!

-Chetan K

Evaluate the piece-defined function for the given of x by matching the domain values with the range values

Answers

Answer:

The correct answer is:

(1) d

(2) b

(3) a

(4) c

(5) e

Step-by-step explanation:

The given problem is incomplete. Find the attachment of the full question.

According to the question,

(1)

By putting x = -2, we get

[tex]f(x)=(-2)^3-5[/tex]

       [tex]=-8-5[/tex]

       [tex]=-13[/tex]

(2)

By putting x = 0, we get

[tex]f(x)=(0)^3-5[/tex]

       [tex]=0-5[/tex]

       [tex]=-5[/tex]

(3)

By putting x = 3, we get

[tex]f(x)=8[/tex]

(4)

By putting x = 9, we get

[tex]f(x)=(2)^{9-7}[/tex]

       [tex]=(2)^2[/tex]

       [tex]=4[/tex]

(5)

By putting x = 11, we get

[tex]f(x)=(2)^{11-7}[/tex]

       [tex]=(2)^4[/tex]

       [tex]=16[/tex]

Which could be the graph of f(x) = |x -h| + k if h and k are both positive?

Answers

Step-by-step explanation:

The graph of the function y=|x| is as shown in the attached diagram. The only possible translation of this graph right and up is option A. (In option B graph is translated right and down, in option C- left and up and in option D - left and down). Answer: correct choice is A.

Answer:

A

Step-by-step explanation:

Just did it

There are 271 songs on Amy's computer. She
wants to make playlists that have 9 songs
each. How many full playlists can she make?

Answers

Answer:

30

Step-by-step explanation:

You do 271 ÷9

= 30.11111111

The answer is your whole number (30)

Monique goes out for coffee once per week. She has already spent $40.86 this year on her coffees and each one is $4.54. There are 27 weeks left in this school year for Monique. If she continues getting coffee at this rate how much money will she have spent on coffee by the end of the year?

Answers

Firstly, 27 x 4.54
= $122.58

$ [tex]$163.44[/tex] money will she have spent on coffee by the end of the year.

What is PEDMAS rule?

PEDMAS rule states that the order of operation starts with the parentheses first or the calculation which is enclosed in brackets. Then the operation is performed on exponents(degree or square roots) and later we do operations on division & multiplication and at last addition and subtraction.

According to the question

Monique goes out for coffee once per week.

She has already spent $40.86 this year on her coffees and each one is $4.54.

There are 27 weeks left in this school year for Monique.

If she continues getting coffee at this rate, money will she have spent on coffee by the end of the year:

Based on the given conditions

[tex]40.86 + 27 \times 4.54[/tex]

By using PEDMAS rule, first we have to multiply 27 and 4.54

= [tex]40.86 + 122.58[/tex]

By using PEDMAS rule, first we have to add 40.86 and 122.58

= [tex]163.44[/tex]

Thus, $163.44 money will she have spent on coffee by the end of the year.

Find out more information about PEDMAS rule here

https://brainly.com/question/24086845

#SPJ3

PLEASE HELP ME I AM SUFFERING GREATLY

Answers

Answer:

6 hours ( i think)

Step-by-step explanation:

new crew = 12 hours, experienced crew = 6 hours

new crew + experienced crew = 12 hours and 6 hours

the help that they get together ; 12-6

Find the measure of the arc or angle indicated.
5)
R
1290
?
S
450
A) 93°
D) 48°
C) 96°
B) 1320
E) 1140

Answers

The answer is D 48 degrees

The manager of a football team says the probability that the team will win the next match is 0.6 and the probability it will lose is is 0.1 Find the probability that the team will: a) not win b) not lose c) not draw

Answers

Answer:

A. 0.4

B.0.9

C.0.7

Step-by-step explanation:

Answers:

a)    0.4b)   0.9c)   0.7

===========================================

Explanation:

Part (a)

P(winning) = 0.6 is given

The probability of not winning is 1 - P(winning) = 1 - 0.6 = 0.4

----------------

Part (b)

P(losing) = 0.1 is given

The probability of not losing is 1 - P(losing) = 1 - 0.1 = 0.9

-----------------

Part (c)

The probability of winning is 0.6 and the probability of losing is 0.1

This adds to 0.6+0.1 = 0.7, which must mean 0.3 is the probability of drawing (since 0.7+0.3 = 1)

So the probability of not drawing is 1 - P(draw) = 1 - 0.3 = 0.7, which is exactly the sum of the probabilities of winning and losing.

Show that √2 is an irrational numbe.​

Answers

Answer:

[tex] \sqrt{2} =1.41421356237...[/tex]

Answer:

The proof that √2 is indeed irrational is usually found in college level math texts, but it isn't that difficult to follow. It does not rely on computers at all, but instead is a "proof by contradiction": if √2 WERE a rational number, we'd get a contradiction.

...

A proof that the square root of 2 is irrational.

2 = (2k)2/b2

b2 = 2k2

What is the solution to the system of equations ?
Y=2/3 x + 3
X= -2

Answers

Answer:

5/3

Step-by-step explanation:

Answer:

x = -2, y = 5/3

Step-by-step explanation:

y = [tex]\frac{2}{3} x+3[/tex]            x = -2

[tex]y=\frac{2}{3} (-2)+3[/tex]

[tex]-\frac{4}{3}[/tex] + 3 =

[tex]-1\frac{1}{3}[/tex] + 3 = [tex]\frac{5}{3}[/tex]

What is the equation of the line

Answers

The answer is
(0,0)
Goodluck

Please help I’ll give brainliest

Answers

Answer: $108199

Step-by-step explanation:

Step 1) Find the deposit cost

35/100 × 89540 = 31339

Step 2) Find the total cost of the monthly payments

2135 × 36 = 76860

Step 3) Find the final cost

31339 + 76860 = $108199

It wants me to use division, but sadly I am not getting the answers I want with it. Please help!

Answers

Answer:

a. 70 gallons of water were leaked each month.

Step-by-step explanation:

Set up an equation:

Variable x = gallons leaked each month

210/3 = x/1

Cross multiply:

210 × 1 = 3 × x

210 = 3x

Divide by 3 to isolate the variable:

70 = x

The division part is only 210 ÷ 3 = x. I wrote it down this way so you can see how I came to this equation.

Check your work:

210/3 = 70/1

210 × 1 = 3 × 70

210 = 210

Correct!

Other Questions
Choose 3 values that would make this inequality true. n - 3 10 1415510221330 how would you explain the concept of literature to a child Calculate the measure of angle P: Which of the following describes sectionalism?the idea that a state could refuse to follow a federal law it disagreed witha strong loyalty to a state or region, sometimes at the expense of a nationthe idea that states have powers separate from the federal governmenta system that divides power between national and state governments How does the author illustrate the importance of radiotelescopes to scientific study?into The reading on your speedometer is theA) average velocityB) none of these is correctC) speedD) instantaneous velocity A 35 kg child slides down a playground slide at a constant speed. The slide has a height of 3.8 m and is 8.0 m long. Find the magnitude of the kinetic friction force acting on the child. find the percent change from 4/5 to 3/5 True or False: It is legal to pass in Florida if you believe it issafe, even if your view is obstructed. Which of the following economies is expected to experience significant growth over the next 40 years?1) The United States 2)China 3)India 4) All choices are correct Someone pls help. We need to find m angle f O verbo regular "turn into", que completa o espao 1 no texto com o Present Perfect, fica escrito * A) turn did B) tarn anto C) turned into B) turning into Colder climates promote higher biodiversity because they are usually larger in size. This provides space for more organisms. What are the possible values of x in x2 + 3x + 3 = 0? in the business world what are people measured by Hallar el menor nmero no divisible por 4, 6, 9, 11 y 12, tal que al dividirlo por estos nmeros se obtengan restos iguales. someone help me on this ixl Please help thanks! Brainliest ExplorersMatch the explorer with their most important accomplishmentSamuel de Champlainexplored North America and the ArcticJames Cookestablished colony of QuebecRobert La Salleexplored MississippiHenry Hudsondiscovered Australia Talk about the weather you experienced on a vacation. Did the weather have a good or bad effect on your plans? Explain